LSAT and Law School Admissions Forum

Get expert LSAT preparation and law school admissions advice from PowerScore Test Preparation.

 Administrator
PowerScore Staff
  • PowerScore Staff
  • Posts: 8917
  • Joined: Feb 02, 2011
|
#23170
Complete Question Explanation

Flaw in the Reasoning. The correct answer choice is (A)

In this stimulus Thomas makes and argument about the Jeffery's right to vote. One little word, "only," makes all the difference. Althea correctly points out the flaw in Thomas' argument with and example and then the question stem asks you to identify that flaw. Thomas says that "only members in good standing may vote" and then demonstrates that Jeffery is in good standing. However, although he has proven that Jeffery is not prohibited from voting, he has not proven that the president had no right to disallow his vote, as the conclusion in the first line states. If the club rules said, "All members in good standing may vote," then Thomas's argument would follow.

Answer choice(A): This is the correct answer choice, because it points out the flaw in Thomas's argument directly. Because he is a member in good standing, Jeffrey is not prohibited from voting for this reason, but that does not mean he has a right to vote. Simply because he is not prohibited for one reason does not mean he is authorized to vote in all circumstances.

Answer choice (B) This would be the correct answer to an ad hominem ("to the person") fallacy, but Thomas does not attack the person of the club president in any way.

Answer choice (C) Thomas does not talk about a statement that is not actually denied. Remember the family to which these types of questions belong: the correct answer choice must be present in the stimulus.

Answer choice (D) This answer choice attempts to lead you astray by pointing out something that the argument does not do. However, just because an answer choice is accurate in terms of what it says, it does not mean that it is correct. It must point out a flaw, and this is not a flaw in the reasoning. It is irrelevant information.

Answer choice (E) We can dismiss this answer choice because it concerns Althea. The question stem asks specifically about Thomas' argument, so any answer choice that involves Althea is automatically incorrect.
 prep88
  • Posts: 37
  • Joined: Jan 20, 2015
|
#19007
Hi,

Can anyone please explain the conditional relationship here and how to diagram it? "only members in good standing may vote"

Thank you!
 David Boyle
PowerScore Staff
  • PowerScore Staff
  • Posts: 836
  • Joined: Jun 07, 2013
|
#19008
prep88 wrote:Hi,

Can anyone please explain the conditional relationship here and how to diagram it? "only members in good standing may vote"

Thank you!
Hello prep88,

"Only members in good standing may vote" would be diagrammed "Vote :arrow: members in good standing". Or for short, "v :arrow: mgs". This is because being a good-standing member is a necessity for voting. (Not the other way around: a member in good standing may not be obliged to vote, but maybe he/she could abstain from voting.)

Hope this helps,
David
 wulflov
  • Posts: 9
  • Joined: Apr 22, 2017
|
#35583
Hi there,

Could somebody provide some more explanation as to why (E) is incorrect? It seems misleading to say "any answer choice that involves Althea is automatically incorrect," since Thomas's argument relies on a premise provided by Althea ("You've admitted that..."). After all, if we had the answer choice "everything Althea says is a lie," it would certainly count against Thomas's argument.

Is it possible I'm misunderstanding what this type of question stem is asking for?
 Kristina Moen
PowerScore Staff
  • PowerScore Staff
  • Posts: 230
  • Joined: Nov 17, 2016
|
#35614
Hi wulflov,

Thomas' argument is flawed, with or without Althea's argument. In fact, Althea actually points out the flaw for us! We are not looking for a flaw in Althea's argument. So any answer choice that involves Althea is NOT describing the flaw in Thomas' argument. His argument stands alone.

This is a Flaw in the Reasoning question, which means the answer choice describes the stimulus. You are not looking to add any information, so an answer choice that says "Everything Althea says is a lie" would not be appropriate here. That would be new information. You are looking for something that describes Thomas' method of reasoning - specifically the flaw in that method of reasoning. Here, he mistakes something that is necessary (being in good standing) to vote as being something sufficient to vote.

It's like if I said that only people who graduate from college can go to law school. Graduating from college is necessary. But it would be FLAW to then infer that Jane can go law school because she graduated from college. As we all know, there's a few more steps she would have to take, like taking the LSAT. :)
 Etsevdos
  • Posts: 62
  • Joined: Oct 22, 2017
|
#41230
I diagrammed this correctly and had a perfect Pre phrase. I could not pull that from a though. The wording confused me. How do you equate A to assumes sufficient = necessary?

Also, I did not read second aregument noting that at best it could only give irrelevant context? Would you recommend this or not?
 nicholaspavic
PowerScore Staff
  • PowerScore Staff
  • Posts: 271
  • Joined: Jun 12, 2017
|
#42081
Hi etsevdos,

I am terribly sorry. I am just not following your questions. Can you tell me what response you are referring to and asking about? I can help you with this question then. Thank you! :-D
 lathlee
  • Posts: 652
  • Joined: Apr 01, 2016
|
#42547
Hi. I got this question correct, but I cann't name the flaw type. may i get the name plz?Is it uncertain term flaw? This is driving me crazy haha
 James Finch
PowerScore Staff
  • PowerScore Staff
  • Posts: 943
  • Joined: Sep 06, 2017
|
#42599
Hi Lathlee,

This question is a False Dilemma, where Thomas is trying to argue that either Jeffrey may vote and have his vote counted, or may not vote at all, ignoring the possibility that the club has other rules that would authorize the club president to disallow Jeffrey's vote, even as a member in good standing.

Hope this helps!
 shop.shim@gmail.com
  • Posts: 3
  • Joined: Dec 20, 2014
|
#43280
According to the stimulus, the club president has disallowed Jeffrey to vote. Thomas is arguing that that was in violation of club rules.

Premise 1: Rule: Vote :arrow: Good standing member
Premise 2: Jeffrey is a good standing member
Conclusion: The president’s action of disallowing Jeffrey to vote was in violation of club rules.

Here are club rules: only good standing members may vote. (vote :arrow: good standing member)
Jeffrey paid his dues on time and therefore he is a good standing member. Necessary condition is satisfied, therefore we don’t know if he is allowed to vote or not allowed to vote. There could be other criteria to qualify to vote that Jeffrey does not meet, in which case he is not allowed to vote. Or Jeffrey meets all the criteria to vote, in which case he is allowed to vote. We have no information about it.

But, Thomas concludes that the president’s action of disallowing Jeffrey to vote was in violation of club rules. Disallowing Jeff's vote could or could not be in violation of the rules. In spite of this possibility that it may not be in violation, Thomas made a determination that it is in violation. This is the flaw. He is saying that allowing him to vote is in compliance with (authorized by) the rule. (In other words, to be in compliance, Jeffrey should be allowed to vote.)

And that is what answer choice (A) is saying. His argument fails to take into account the distinction between something not being prohibited (allowing Jeffrey to vote) and its being authorized (in compliance with the rule). He is saying that allowing to vote is in compliance with (authorized by) the rule. At the end of the answer choice A, words “by the rule” is omitted. “Authorized” here does not mean his being authorized to vote, but rather, it means being authorized by the rule.

We can also view it as sufficient condition, necessary condition confusion flaw. Because in reaching the conclusion, Thomas mistakenly assumed the club rule as: good standing member :arrow: vote. But this is not in any of the answer choices.

Get the most out of your LSAT Prep Plus subscription.

Analyze and track your performance with our Testing and Analytics Package.